categories - Category Theory list
 help / color / mirror / Atom feed
* Re: Re: stupid question?
@ 2000-03-29 23:13 Max Kanovitch
  2000-03-30  3:00 ` Todd Wilson
  0 siblings, 1 reply; 5+ messages in thread
From: Max Kanovitch @ 2000-03-29 23:13 UTC (permalink / raw)
  To: categories

 Dear M.M. Mawanda,
 
 > >I have been asked the following question: Is it true that any function
 > >defined in a real number closed interval [a,b] (there is not a hypothesis
 > >of continuity) is bounded in an open  subinterval (c,d) of [a,b]?

 The real fun is about a function f such that
 f is unbounded in any open interval (c,d), and
 in addition to that:     f(x+y) = f(x)+f(y).
 
 > Date: Wed, 29 Mar 2000 15:23:16 -0500 (EST)
 > From: Peter Freyd <pjf@saul.cis.upenn.edu>
 > Subject: categories: Re: stupid question?
 > 
 >  M.M. Mawanda asks:
 > 
 > >I have been asked the following question: Is it true that any function
 > >defined in a real number closed interval [a,b] (there is not a hypothesis
 > >of continuity) is bounded in an open  subinterval (c,d) of [a,b]? My
 > >spontaneous was NO. Unfortunately I cannot find a counter-example to
 > >disapproved my answer. Can someone help.     
 > 
 > No it is not true. For example, the function defined by:
 > 
 >   f(x) =  if  x  is irrational then  0  else
 >           if  x =  p/q  where  p  and  q  are co-prime then  q.
 > 



^ permalink raw reply	[flat|nested] 5+ messages in thread

* Re: stupid question?
  2000-03-29 23:13 Re: stupid question? Max Kanovitch
@ 2000-03-30  3:00 ` Todd Wilson
  0 siblings, 0 replies; 5+ messages in thread
From: Todd Wilson @ 2000-03-30  3:00 UTC (permalink / raw)
  To: categories

On Wed, 29 Mar 2000, Max Kanovitch wrote:
>  The real fun is about a function f such that
>  f is unbounded in any open interval (c,d), and
>  in addition to that:     f(x+y) = f(x)+f(y).

Using AC/Zorn's Lemma, we can construct 2^(2^Aleph_0) such functions,
as follows.  Let B be a basis for the reals R as a rational vector
space.  Clearly, |B| = 2^Aleph_0.  For any non-empty proper subset C
of B, let g be its characteristic function (g(x)=1 if x in C, g(x)=0
otherwise) and let f be the unique linear extension of g to R.  Then f
is linear, and its graph is dense in R^2, since, if c and d are such
that g(c)=1 and g(d)=0, then f(qc+rd) = q for all rationals q,r, and r
can be varied to make qc+rd as close as desired to any given real.

-- 
Todd Wilson
Computer Science Department
California State University, Fresno



^ permalink raw reply	[flat|nested] 5+ messages in thread

* RE: stupid question?
@ 2000-03-29 21:48 Wendt, Michael - SSMD/DMES
  0 siblings, 0 replies; 5+ messages in thread
From: Wendt, Michael - SSMD/DMES @ 2000-03-29 21:48 UTC (permalink / raw)
  To: categories

How about the following?

f(x) = 0 if x is irrational and

f(a/b) = a, where a/b is a fraction in lowest terms

Certainly within any open interval, there are rationals of arbitrarily large
numerator.

For a function that is not bounded above or below, how about:

f(x) = 0 if x is irrational

f(a/b) = a if b is even

f(a/b) = -a if b is odd

-----Original Message-----
From: M.M. Mawanda [mailto:mm.mawanda@nul.ls]
Sent: March 29,2000 4:08 PM
To: cat-dist@mta.ca
Subject: categories: stupid question?


I have been asked the following question: Is it true that any function
defined in a real number closed interval [a,b] (there is not a hypothesis
of continuity) is bounded in an open  subinterval (c,d) of [a,b]? My
spontaneous was NO. Unfortunately I cannot find a counter-example to
disapproved my answer. Can someone help.     






^ permalink raw reply	[flat|nested] 5+ messages in thread

* Re: stupid question?
@ 2000-03-29 20:23 Peter Freyd
  0 siblings, 0 replies; 5+ messages in thread
From: Peter Freyd @ 2000-03-29 20:23 UTC (permalink / raw)
  To: categories; +Cc: mm.mawanda

 M.M. Mawanda asks:

>I have been asked the following question: Is it true that any function
>defined in a real number closed interval [a,b] (there is not a hypothesis
>of continuity) is bounded in an open  subinterval (c,d) of [a,b]? My
>spontaneous was NO. Unfortunately I cannot find a counter-example to
>disapproved my answer. Can someone help.     

No it is not true. For example, the function defined by:

  f(x) =  if  x  is irrational then  0  else
          if  x =  p/q  where  p  and  q  are co-prime then  q.



^ permalink raw reply	[flat|nested] 5+ messages in thread

* stupid question?
@ 2000-03-29 17:36 M.M. Mawanda
  0 siblings, 0 replies; 5+ messages in thread
From: M.M. Mawanda @ 2000-03-29 17:36 UTC (permalink / raw)
  To: cat-dist

I have been asked the following question: Is it true that any function
defined in a real number closed interval [a,b] (there is not a hypothesis
of continuity) is bounded in an open  subinterval (c,d) of [a,b]? My
spontaneous was NO. Unfortunately I cannot find a counter-example to
disapproved my answer. Can someone help.     





^ permalink raw reply	[flat|nested] 5+ messages in thread

end of thread, other threads:[~2000-03-30  3:00 UTC | newest]

Thread overview: 5+ messages (download: mbox.gz / follow: Atom feed)
-- links below jump to the message on this page --
2000-03-29 23:13 Re: stupid question? Max Kanovitch
2000-03-30  3:00 ` Todd Wilson
  -- strict thread matches above, loose matches on Subject: below --
2000-03-29 21:48 Wendt, Michael - SSMD/DMES
2000-03-29 20:23 Peter Freyd
2000-03-29 17:36 M.M. Mawanda

This is a public inbox, see mirroring instructions
for how to clone and mirror all data and code used for this inbox;
as well as URLs for NNTP newsgroup(s).